How to decide convergence of IntegralsLooking for the recurrence relation for certain trigonometric integralsEvaluating integrals with trigonometric functionHow to find the indefinite Integral of ln|sec|?Convergence of Riemann sums for improper integralsShowing convergence/divergence of generalized integralsCan definite integrals be indeterminate?Whats wrong in this approach to evaluate $int_0^fracpi2 fracsin xcos xdxsin x+cos x$Is there a non-trivial definite integral that values to $fracepi$?Evaluate Integrals Using Convergence TheoremsExamples of use of integrals to show convergence of a limit

Can a Gentile theist be saved?

How to prevent YouTube from showing already watched videos?

For airliners, what prevents wing strikes on landing in bad weather?

Science Fiction story where a man invents a machine that can help him watch history unfold

How can I raise concerns with a new DM about XP splitting?

Is there enough fresh water in the world to eradicate the drinking water crisis?

Proof of Lemma: Every integer can be written as a product of primes

Latex for-and in equation

Lightning Web Component - do I need to track changes for every single input field in a form

How to color a zone in Tikz

How to check participants in at events?

What to do when my ideas aren't chosen, when I strongly disagree with the chosen solution?

Can somebody explain Brexit in a few child-proof sentences?

Freedom of speech and where it applies

Would it be legal for a US State to ban exports of a natural resource?

How to deal with or prevent idle in the test team?

What is the term when two people sing in harmony, but they aren't singing the same notes?

Calculating the number of days between 2 dates in Excel

Organic chemistry Iodoform Reaction

Is there an wasy way to program in Tikz something like the one in the image?

Can I Retrieve Email Addresses from BCC?

I2C signal and power over long range (10meter cable)

My boss asked me to take a one-day class, then signs it up as a day off

Simulating a probability of 1 of 2^N with less than N random bits



How to decide convergence of Integrals


Looking for the recurrence relation for certain trigonometric integralsEvaluating integrals with trigonometric functionHow to find the indefinite Integral of ln|sec|?Convergence of Riemann sums for improper integralsShowing convergence/divergence of generalized integralsCan definite integrals be indeterminate?Whats wrong in this approach to evaluate $int_0^fracpi2 fracsin xcos xdxsin x+cos x$Is there a non-trivial definite integral that values to $fracepi$?Evaluate Integrals Using Convergence TheoremsExamples of use of integrals to show convergence of a limit













5












$begingroup$


I got this doubt while evaluating the integrals:



$$I=int_0^fracpi2ln(sin x)sin xdx$$ and



$$J=int_0^fracpi4csc xdx$$



Now even though the integrand $f(x)=ln(sin x)sin x$ is not defined at $x=0$ which is the lower limit, still it has a finite answer.



But integrand in $J$ is not defined at $x=0$ and integral is infinite.



So how to identify without explicitly evaluating?










share|cite|improve this question









$endgroup$











  • $begingroup$
    Comparison test is definite a way to go.
    $endgroup$
    – Sangchul Lee
    yesterday















5












$begingroup$


I got this doubt while evaluating the integrals:



$$I=int_0^fracpi2ln(sin x)sin xdx$$ and



$$J=int_0^fracpi4csc xdx$$



Now even though the integrand $f(x)=ln(sin x)sin x$ is not defined at $x=0$ which is the lower limit, still it has a finite answer.



But integrand in $J$ is not defined at $x=0$ and integral is infinite.



So how to identify without explicitly evaluating?










share|cite|improve this question









$endgroup$











  • $begingroup$
    Comparison test is definite a way to go.
    $endgroup$
    – Sangchul Lee
    yesterday













5












5








5





$begingroup$


I got this doubt while evaluating the integrals:



$$I=int_0^fracpi2ln(sin x)sin xdx$$ and



$$J=int_0^fracpi4csc xdx$$



Now even though the integrand $f(x)=ln(sin x)sin x$ is not defined at $x=0$ which is the lower limit, still it has a finite answer.



But integrand in $J$ is not defined at $x=0$ and integral is infinite.



So how to identify without explicitly evaluating?










share|cite|improve this question









$endgroup$




I got this doubt while evaluating the integrals:



$$I=int_0^fracpi2ln(sin x)sin xdx$$ and



$$J=int_0^fracpi4csc xdx$$



Now even though the integrand $f(x)=ln(sin x)sin x$ is not defined at $x=0$ which is the lower limit, still it has a finite answer.



But integrand in $J$ is not defined at $x=0$ and integral is infinite.



So how to identify without explicitly evaluating?







integration algebra-precalculus convergence






share|cite|improve this question













share|cite|improve this question











share|cite|improve this question




share|cite|improve this question










asked yesterday









Umesh shankarUmesh shankar

3,05931220




3,05931220











  • $begingroup$
    Comparison test is definite a way to go.
    $endgroup$
    – Sangchul Lee
    yesterday
















  • $begingroup$
    Comparison test is definite a way to go.
    $endgroup$
    – Sangchul Lee
    yesterday















$begingroup$
Comparison test is definite a way to go.
$endgroup$
– Sangchul Lee
yesterday




$begingroup$
Comparison test is definite a way to go.
$endgroup$
– Sangchul Lee
yesterday










2 Answers
2






active

oldest

votes


















6












$begingroup$

Notice, however, that



begineqnarray
lim_xto0^+ln(sin x)sin x&=&lim_xto0^+fracln(sin x)csc x\
&=&lim_xto0^+fraccot x(-csc xcot x)\
&=&lim_xto0^+(-sin x)\
&=&0
endeqnarray



Here is the graph of $y=ln(sin x)sin x$



graph of ln(sin x)(sin x)






share|cite|improve this answer











$endgroup$












  • $begingroup$
    So does it mean if limit is finite at the end points, integral exists?
    $endgroup$
    – Umesh shankar
    yesterday










  • $begingroup$
    In this case, the discontinuity at $x=0$ was a removable discontinuity. But such is not the case at $x=0$ for $csc x$ because of the vertical asymptote. However, even in the case where there is a vertical asymptote the integral may still converge. For example, $int_0^1dfrac1sqrtx,dx$.
    $endgroup$
    – John Wayland Bales
    yesterday



















0












$begingroup$

For the second integral $J$, the integrand goes to $infty$ as $xto 0$. Roughly $frac1sin x = frac1x$. From the examples $int_0^1 x^-a dx$ we know that $a=1$ is divergent, albeit borderline so. Since
$$
sin x = x - frac13!x^3 pm ...
$$

we have $sin x leq x$ for $x>0$ small (from elementary trigonometry, or since the tail of a (edit: convergent) alternating series with terms of decreasing value is dominated by any previous term). Thus $frac1sin x geq frac1x$ and the integral $J$ is divergent.






share|cite|improve this answer











$endgroup$












    Your Answer





    StackExchange.ifUsing("editor", function ()
    return StackExchange.using("mathjaxEditing", function ()
    StackExchange.MarkdownEditor.creationCallbacks.add(function (editor, postfix)
    StackExchange.mathjaxEditing.prepareWmdForMathJax(editor, postfix, [["$", "$"], ["\\(","\\)"]]);
    );
    );
    , "mathjax-editing");

    StackExchange.ready(function()
    var channelOptions =
    tags: "".split(" "),
    id: "69"
    ;
    initTagRenderer("".split(" "), "".split(" "), channelOptions);

    StackExchange.using("externalEditor", function()
    // Have to fire editor after snippets, if snippets enabled
    if (StackExchange.settings.snippets.snippetsEnabled)
    StackExchange.using("snippets", function()
    createEditor();
    );

    else
    createEditor();

    );

    function createEditor()
    StackExchange.prepareEditor(
    heartbeatType: 'answer',
    autoActivateHeartbeat: false,
    convertImagesToLinks: true,
    noModals: true,
    showLowRepImageUploadWarning: true,
    reputationToPostImages: 10,
    bindNavPrevention: true,
    postfix: "",
    imageUploader:
    brandingHtml: "Powered by u003ca class="icon-imgur-white" href="https://imgur.com/"u003eu003c/au003e",
    contentPolicyHtml: "User contributions licensed under u003ca href="https://creativecommons.org/licenses/by-sa/3.0/"u003ecc by-sa 3.0 with attribution requiredu003c/au003e u003ca href="https://stackoverflow.com/legal/content-policy"u003e(content policy)u003c/au003e",
    allowUrls: true
    ,
    noCode: true, onDemand: true,
    discardSelector: ".discard-answer"
    ,immediatelyShowMarkdownHelp:true
    );



    );













    draft saved

    draft discarded


















    StackExchange.ready(
    function ()
    StackExchange.openid.initPostLogin('.new-post-login', 'https%3a%2f%2fmath.stackexchange.com%2fquestions%2f3160811%2fhow-to-decide-convergence-of-integrals%23new-answer', 'question_page');

    );

    Post as a guest















    Required, but never shown

























    2 Answers
    2






    active

    oldest

    votes








    2 Answers
    2






    active

    oldest

    votes









    active

    oldest

    votes






    active

    oldest

    votes









    6












    $begingroup$

    Notice, however, that



    begineqnarray
    lim_xto0^+ln(sin x)sin x&=&lim_xto0^+fracln(sin x)csc x\
    &=&lim_xto0^+fraccot x(-csc xcot x)\
    &=&lim_xto0^+(-sin x)\
    &=&0
    endeqnarray



    Here is the graph of $y=ln(sin x)sin x$



    graph of ln(sin x)(sin x)






    share|cite|improve this answer











    $endgroup$












    • $begingroup$
      So does it mean if limit is finite at the end points, integral exists?
      $endgroup$
      – Umesh shankar
      yesterday










    • $begingroup$
      In this case, the discontinuity at $x=0$ was a removable discontinuity. But such is not the case at $x=0$ for $csc x$ because of the vertical asymptote. However, even in the case where there is a vertical asymptote the integral may still converge. For example, $int_0^1dfrac1sqrtx,dx$.
      $endgroup$
      – John Wayland Bales
      yesterday
















    6












    $begingroup$

    Notice, however, that



    begineqnarray
    lim_xto0^+ln(sin x)sin x&=&lim_xto0^+fracln(sin x)csc x\
    &=&lim_xto0^+fraccot x(-csc xcot x)\
    &=&lim_xto0^+(-sin x)\
    &=&0
    endeqnarray



    Here is the graph of $y=ln(sin x)sin x$



    graph of ln(sin x)(sin x)






    share|cite|improve this answer











    $endgroup$












    • $begingroup$
      So does it mean if limit is finite at the end points, integral exists?
      $endgroup$
      – Umesh shankar
      yesterday










    • $begingroup$
      In this case, the discontinuity at $x=0$ was a removable discontinuity. But such is not the case at $x=0$ for $csc x$ because of the vertical asymptote. However, even in the case where there is a vertical asymptote the integral may still converge. For example, $int_0^1dfrac1sqrtx,dx$.
      $endgroup$
      – John Wayland Bales
      yesterday














    6












    6








    6





    $begingroup$

    Notice, however, that



    begineqnarray
    lim_xto0^+ln(sin x)sin x&=&lim_xto0^+fracln(sin x)csc x\
    &=&lim_xto0^+fraccot x(-csc xcot x)\
    &=&lim_xto0^+(-sin x)\
    &=&0
    endeqnarray



    Here is the graph of $y=ln(sin x)sin x$



    graph of ln(sin x)(sin x)






    share|cite|improve this answer











    $endgroup$



    Notice, however, that



    begineqnarray
    lim_xto0^+ln(sin x)sin x&=&lim_xto0^+fracln(sin x)csc x\
    &=&lim_xto0^+fraccot x(-csc xcot x)\
    &=&lim_xto0^+(-sin x)\
    &=&0
    endeqnarray



    Here is the graph of $y=ln(sin x)sin x$



    graph of ln(sin x)(sin x)







    share|cite|improve this answer














    share|cite|improve this answer



    share|cite|improve this answer








    edited yesterday

























    answered yesterday









    John Wayland BalesJohn Wayland Bales

    15.1k21238




    15.1k21238











    • $begingroup$
      So does it mean if limit is finite at the end points, integral exists?
      $endgroup$
      – Umesh shankar
      yesterday










    • $begingroup$
      In this case, the discontinuity at $x=0$ was a removable discontinuity. But such is not the case at $x=0$ for $csc x$ because of the vertical asymptote. However, even in the case where there is a vertical asymptote the integral may still converge. For example, $int_0^1dfrac1sqrtx,dx$.
      $endgroup$
      – John Wayland Bales
      yesterday

















    • $begingroup$
      So does it mean if limit is finite at the end points, integral exists?
      $endgroup$
      – Umesh shankar
      yesterday










    • $begingroup$
      In this case, the discontinuity at $x=0$ was a removable discontinuity. But such is not the case at $x=0$ for $csc x$ because of the vertical asymptote. However, even in the case where there is a vertical asymptote the integral may still converge. For example, $int_0^1dfrac1sqrtx,dx$.
      $endgroup$
      – John Wayland Bales
      yesterday
















    $begingroup$
    So does it mean if limit is finite at the end points, integral exists?
    $endgroup$
    – Umesh shankar
    yesterday




    $begingroup$
    So does it mean if limit is finite at the end points, integral exists?
    $endgroup$
    – Umesh shankar
    yesterday












    $begingroup$
    In this case, the discontinuity at $x=0$ was a removable discontinuity. But such is not the case at $x=0$ for $csc x$ because of the vertical asymptote. However, even in the case where there is a vertical asymptote the integral may still converge. For example, $int_0^1dfrac1sqrtx,dx$.
    $endgroup$
    – John Wayland Bales
    yesterday





    $begingroup$
    In this case, the discontinuity at $x=0$ was a removable discontinuity. But such is not the case at $x=0$ for $csc x$ because of the vertical asymptote. However, even in the case where there is a vertical asymptote the integral may still converge. For example, $int_0^1dfrac1sqrtx,dx$.
    $endgroup$
    – John Wayland Bales
    yesterday












    0












    $begingroup$

    For the second integral $J$, the integrand goes to $infty$ as $xto 0$. Roughly $frac1sin x = frac1x$. From the examples $int_0^1 x^-a dx$ we know that $a=1$ is divergent, albeit borderline so. Since
    $$
    sin x = x - frac13!x^3 pm ...
    $$

    we have $sin x leq x$ for $x>0$ small (from elementary trigonometry, or since the tail of a (edit: convergent) alternating series with terms of decreasing value is dominated by any previous term). Thus $frac1sin x geq frac1x$ and the integral $J$ is divergent.






    share|cite|improve this answer











    $endgroup$

















      0












      $begingroup$

      For the second integral $J$, the integrand goes to $infty$ as $xto 0$. Roughly $frac1sin x = frac1x$. From the examples $int_0^1 x^-a dx$ we know that $a=1$ is divergent, albeit borderline so. Since
      $$
      sin x = x - frac13!x^3 pm ...
      $$

      we have $sin x leq x$ for $x>0$ small (from elementary trigonometry, or since the tail of a (edit: convergent) alternating series with terms of decreasing value is dominated by any previous term). Thus $frac1sin x geq frac1x$ and the integral $J$ is divergent.






      share|cite|improve this answer











      $endgroup$















        0












        0








        0





        $begingroup$

        For the second integral $J$, the integrand goes to $infty$ as $xto 0$. Roughly $frac1sin x = frac1x$. From the examples $int_0^1 x^-a dx$ we know that $a=1$ is divergent, albeit borderline so. Since
        $$
        sin x = x - frac13!x^3 pm ...
        $$

        we have $sin x leq x$ for $x>0$ small (from elementary trigonometry, or since the tail of a (edit: convergent) alternating series with terms of decreasing value is dominated by any previous term). Thus $frac1sin x geq frac1x$ and the integral $J$ is divergent.






        share|cite|improve this answer











        $endgroup$



        For the second integral $J$, the integrand goes to $infty$ as $xto 0$. Roughly $frac1sin x = frac1x$. From the examples $int_0^1 x^-a dx$ we know that $a=1$ is divergent, albeit borderline so. Since
        $$
        sin x = x - frac13!x^3 pm ...
        $$

        we have $sin x leq x$ for $x>0$ small (from elementary trigonometry, or since the tail of a (edit: convergent) alternating series with terms of decreasing value is dominated by any previous term). Thus $frac1sin x geq frac1x$ and the integral $J$ is divergent.







        share|cite|improve this answer














        share|cite|improve this answer



        share|cite|improve this answer








        edited yesterday

























        answered yesterday









        user626368user626368

        193




        193



























            draft saved

            draft discarded
















































            Thanks for contributing an answer to Mathematics Stack Exchange!


            • Please be sure to answer the question. Provide details and share your research!

            But avoid


            • Asking for help, clarification, or responding to other answers.

            • Making statements based on opinion; back them up with references or personal experience.

            Use MathJax to format equations. MathJax reference.


            To learn more, see our tips on writing great answers.




            draft saved


            draft discarded














            StackExchange.ready(
            function ()
            StackExchange.openid.initPostLogin('.new-post-login', 'https%3a%2f%2fmath.stackexchange.com%2fquestions%2f3160811%2fhow-to-decide-convergence-of-integrals%23new-answer', 'question_page');

            );

            Post as a guest















            Required, but never shown





















































            Required, but never shown














            Required, but never shown












            Required, but never shown







            Required, but never shown

































            Required, but never shown














            Required, but never shown












            Required, but never shown







            Required, but never shown







            Popular posts from this blog

            Sum ergo cogito? 1 nng

            419 nièngy_Soadمي 19bal1.5o_g

            Queiggey Chernihivv 9NnOo i Zw X QqKk LpB